You are on page 1of 5

MAS 5215/001 Number Theory, Spring 2015

Test 3 (60 minutes)


Date: 04/23/2015
Name:
Show ALL steps.
(1 pt for writing your name nicely)
50 points equal 100%.
Question 1. (5+5 points) Assume that f, F are
P arithmetic functions.P
(a) Assume that n 1. Prove
that
if
F
(n)
=
d|n f (d), then f (n) =
d|n (d)F (n/d).
P
Solution. Assume F (n) = d|n f (d). Then
X

(d)F (n/d) =

d|n

(d)

d|n

X
d |

XX

f (d )

n
d

(d)f (d )

d |n d| n
d

(n/d )f (d )

d |n

= f (n).
P
P
(b) Assume that n 1. Prove
that
if
f
(n)
=
(n/d)F
(d),
then
F
(n)
=
d|n
d|n f (d).
P
Solution. Assume f (n) = d|n (n/d)F (d). Then
X

f (d) =

XX

(d/d )F (d )

XX

(d )F (d ),

d|n

d|n d |d

changing variable: d = d d

d |n d | n
d

(n/d )F (d )

d |n

= F (n).

Keeping Scores:
Question Bonus
Score
Out of
1

Total

10

10

13

11

58

Question 2. (4+3+3 points) (a) Let f, g be arithmetic functions. Prove that


Df g (s) = Df (s)Dg (s).
Solution. We have
Df g (s) =

ns

n1

f (d)g(n/d),

using n = dd

d|n

XX

(dd )s f (d)g(d )

d1 d 1

= Df (s)Dg (s).
(b) Show that the Dirichlet series Dk of k is
Dk (s) = (s k).
Solution. Observe that
Dk (s) =

ns k (n) =

n1

ns nk =

n1

n(sk) = (s k).

n1

(c) Determine the Dirichlet series Dk .


Solution. We note first that
X
X
k (n) =
dk 1 =
k (d)0 (n/d) = (k 0 )(n).
d|n

d|n

Hence, we have from parts (a) and (b) that


Dk (s) = Dk (s)D0 (s) = (s k)(s).

QuestionP3. (4+4 points) (a) Let f be a multiplicative function with f (1) = 1. Define
F (n) = d|n (d)f (d). Prove that
Y
F (n) =
(1 f (p)).
p|n

P
Solution. Let F (n) = d|n (d)f (d). Consider F (pa ), where p is prime and a is a positive
integer. Then since (1) = 1, (p) = 1 and (pi ) = 0 for all i 2, we have
a

F (p ) =

X
d|pa

(d)f (d) =

a
X

(pi )f (pi ) = (1)f (1) + (p)f (p) = 1 f (p).

i=0

Also, since both and f are multiplicative functions, the function F (n) =
also multiplicative. Thus
Y
Y
F (n) =
F (pep ) =
(1 f (p)).
p|n

p|n

d|n

(d)f (d) is

P
(b) Let F be the arithmetic function given by F (n) = d|n (d) (d). Show that F (n) =
(1)(n) .
Solution. We apply part (a), with f = . We note that for prime p, we have (p) = 2. Then
for n 1, we have
Y
F (n) =
(1 (p)) = (1)(n) .
p|n

(5 points) The CRT says that for a, b Z, x a (mod 11) and x b


Question 4.
(mod 13) iff x 78a 77b (mod 143). Find all x so that x2 126 (mod 143).
Solution. Note that x2 126 (mod 143) is equivalent to
x2 126 5 16

(mod 11),

and

x 126 9

(mod 13).

The above is equivalent to


x 4

(mod 11),

and

x 3

(mod 13).

The CRT now gives


x 81,

x 114

(mod 143).


Question 5. (4+5+4 points) (a) Let p = 131 which is prime and a = 59. Compute ap .
Solution. There are many ways to compute the value. For example,
 
 
 
 
 
 
 
131
13
59
7
6
1
59
=
=
=
=
=
=
= 1.
131
59
59
13
13
7
7

(b) Let p be prime satisfying p 3 (mod 4) and a Z satisfying ap = 1. Show that
x a(p+1)/4 is a square
root of a modulo p.

Solution. Since ap = 1, we have from Eulers Criterion that
 
a
1=
a(p1)/2
p

(mod p).

Thus, for x a(p+1)/4 ,


x2 a(p+1)/2 = a a(p1)/2 a (mod p).
(c) Find all incongruent square roots of a = 59 modulo p = 131. (Note that a32 46
(mod p)).
Solution. From (b), we see that
x a(p+1)/4 = a33 = 46 59 94

(mod p),

is a square root of 59 modulo p. Since p = 131 is prime and a is a quadratic residue, there
are two incongruent square roots of 59 modulo p: 94.

Question 6. (2+5+4 points) (a) State carefully Eulers Criterion.


Solution. Assume that p is an odd prime and a is an integer with p a. Then
(mod p).
(b) Prove Eulers Criterion.
Solution. There are two cases. (I) If
and p x (because p a). Then
a(p1)/2 x2

a
p

a
p

a(p1)/2

= 1, then there is x Z so that x2 a (mod p)

(p1)/2

= xp1 1

(mod p),

where the last congruence follows from FLT.


(II) If ap = 1, then there does not exist x so that x2 a (mod p). We note first that
for each x satisfying 1 x p 1, there is y ax (mod p) so that xy a (mod p).
As a is a quadratic nonresidue, we see that x 6 y (mod p). This means that the integers
1, 2, . . . , p 1 can be paired as {x, y} so that xy a (mod p). (To prove this rigorously, one
would want to show that the mapping f : [p 1] [p 1] defined by f (x) = ax mod p
satisfies f (x) 6= x and f (f (x)) = x for all x [p 1].) Then
(1)(2) (p 1) a(p1)/2

(mod p).

Wilson Theorem gives that (p 1)! 1 (mod p), and hence a(p1)/2 1 (mod p).

(c) Assume that p is an odd prime. Prove that 1
= 1 iff p 1 (mod 4).
p
Solution. Since p is an odd prime, we see that p 1, 3 (mod 4). If p = 4k + 1 for some k,
then
 
1
(1)(p1)/2 = (1)2k = 1 (mod p).
p


1
Since 1
=
1
and
p

3,
we
see
that
= 1 in this case. If p = 4k + 3 for some k, then
p
p


and we have

1
p

1
p

(1)(p1)/2 = (1)2k+1 = 1

= 1 in this case.

(mod p),

The following notation and results can be useful in this exam.


Q
We use n = p|n pep to represent the prime power factorization of n.

The functions , k , k , where k 0, and are defined as follows. (a)P(n) = 1 if


n = 1, and (n) = 0 if n 2. (b) k (n) = nk for n Z+ . (c) k (n) = d|n dk . (d)
= 0 . (e) (n) equals the number of distinct prime divisors of n for n 1.
P
The Mobius function is , and it satisfies d|n (d) = (n).
The Dirichlet convolution of arithmetic functions f and g is the function f g given
by
X
(f g)(n) =
f (d)g(n/d),
n 1.
d|n

The Dirichlet series of an arithmetic function f is Df and is given by


X
Df (s) =
ns f (n),
for values of s whenever the series is convergent.
n1

Let f, F be arithmetic functions satisfying F (n) =


multiplicative iff f is multiplicative.

d|n

f (d). It is known that F is

You might also like